ln√(x2 y2)=arctany x

来源:学生作业帮助网 编辑:作业帮 时间:2024/04/26 20:28:04
设y=ln ln ln x,求y’

y'=(lnlnx)'/lnlnx=(lnx)'/lnxlnlnx=1/xlnxlnlnx

y=ln(ln√x)的导数

你做的是对的y=ln(ln√x)y`=[lh(ln√x)]`=1/ln√x*(ln√x)`=(1/ln√x)*(1/√x)*(√x)`=(1/ln√x)*(1/√x)*(1/2)/√x=1/(2xln

已知x2y2-20xy+x2+81=0求x,y的值

x2y2-20xy+x2+81=(xy-10)2+x2-19=0则xy-10=0且x2-19=0得x=+-根号19y=+-10/根号19对于像这种未知数个数多于方程类型的式子,如果能求解,只有一种情况

反三角函数由于arcTanx在(-π/2,π/2)上是增函数,应该可以得到arcTanx1+arcTanx2=arcTa

arctanx表示一个角度,a1=arctanx1;a2=arctanx2;tana1=x1;tana2=x2;tan(arctanx1+arctanx2)=[tan(a1)+tan(a2)]/(1-

y=ln[ln(ln x)] 求导

复合函数f(x)=lnxg(x)=ln[ln(x)]r(x)=ln{lnln(x)]}r'(x)=[1/lnln(x)]g'(x)=[1/lnln(x)][1/ln(x)]f'(x)=[1/lnln(

ln√X求导

(ln√x)'=1/√x*(√x)'=1/√x*1/(2√x)=1/(2x)

已知x-y=1/xy=3.求x3次方y-2x2y2+xy3

x3次方y-2x2y2+xy3=xy(x²-2xy+y²)=xy(x-y)²=3x3²=27如果本题有什么不明白可以追问,再问:=xy(x2-2xy+y2)=x

求y=Ln(Ln(Ln x))的导数

y=(ln(ln(x))'/ln(ln(x))=(ln(x))'/(ln(x)(ln(ln(x)))=1/(xln(x)ln(ln(x)))

有这样一道题,计算(2x4-4x3y-x2y2)-2(x4-2x3y-y3)+x2y2的值,其中x=0.25,y=-1;

(2x4-4x3y-x2y2)-2(x4-2x3y-y3)+x2y2=2x4-4x3y-x2y2-2x4+4x3y+2y3+x2y2=2y3,因为化简的结果中不含x,所以原式的值与x值无关.

已知x2y2+x2+4xy+13=6x,求x、y的值.

x2y2+4xy+4+x2-6x+9=0,(xy+2)2+(x-3)2=0,∵(xy+2)2≥0,(x-3)2≥0,∴xy+2=0,x-3=0,∴xy=-2,x=3.将x=3代入xy=-2中,解得y=

点(x1y1),(x2y2)在反比例函数y=k/x的图像上,当x1

由题意在y=k/x的图像上的两点(x1,y1)(x2,y2),当x1<x2<0,y1<y2,则可知,y=k/x的图像在第二象限,y随x增大而增大.所以k<0..

双曲线x²/a²-y²/b²=1(a>0,b>0)两条渐近线的夹角为2arcta

即一条渐近线的倾斜角θ是arctan3/4或π/2-arctan3/4所以tanθ=3/4或4/3其中第二个即和y轴夹角是arctan3/4此时两条渐近线的夹角也是2arctan3/4,

y2+x2=2x,则x2y2的范围?

解题思路:圆的参数方程解题过程:varSWOC={};SWOC.tip=false;try{SWOCX2.OpenFile("http://dayi.prcedu.com/include/readq.

(2x2y-2xy2)-[(-3x2y2+3x2y)+(3x2y2-3xy2)],其中x=-1,y=2.

原式=2x2y-2xy2-[-3x2y2+3x2y+3x2y2-3xy2]=2x2y-2xy2+3x2y2-3x2y-3x2y2+3xy2=2x2y-3x2y-2xy2+3xy2+3x2y2-3x2y

已知x+y=4,xy=2,则x3y+x2y2+xy3的值:

x+y=4,xy=2后者平方后二式相加再加后者平方

求导数 y=ln[ln(lnx)]

y=ln(1-x)y'=[1/(1-x)]*(1-x)'=-1/(1-x)14382希望对你有帮助!

化简求值(2X2-2y2)-3(X2y2+X2)+3(X2y2+y2),其中X=-1,y=2

(2X²-2y²)-3(X²y²+X²)+3(X²y²+y²)=2x²-2y²-3x²y&

已知x-y=l,xy=2,求x3y-2x2y2+xy3的值.

∵x-y=l,xy=2,∴x3y-2x2y2+xy3=xy(x2-2xy+y2)=xy(x-y)2=2×1=2.

arctan(tan5π/4)为什么等于arctan(-1) tan(5π/4)=tan(π+π/4)不应等于arcta

你说的对确实是arctan1再问:可参考书上写的是这个问一下谢谢再答:显然,书上印错了